Angular Acceleration of a Rod in an Uneven String Setup

In summary, the conversation discusses the equations and constraints for a rod attached to two strings at unequal lengths and at an angle, and then a modified setup with both strings vertical. The equations for both setups are provided, and there is a discussion about the movement of the rod's center of mass and left end following the snap of one of the strings.
  • #1
Vibhor
971
40

Homework Statement



?temp_hash=4882fd61b6534ddf43b92f7e90f1d4d6.png


Note : In the above setup the string lengths are unequal and the left angle is 30° and right angle is 60° .

Homework Equations

The Attempt at a Solution


[/B]
Just after the string is cut , writing force eq. for rod in vertical direction .

##Mg - Tcos60° = Ma_y ## (1)

Writing force eq in horizontal direction

##Tsin60° = Ma_x## (2)

Writing torque eq about the CM of the rod ,

##Tcos 60°\frac{L}{2} = \frac{ML^2}{12} \alpha ## (3)Writing constraint eq. for the left end point of the rod where string is attached ,the component of acceleration of left end along the string length should be zero .

##a_ycos60° - \frac{αL}{2}cos60° - a_xcos30°= 0 ## (4)Solving the above four eqs .give ##T= \frac{2}{7}Mg##
and ##\alpha = \frac{6g}{7L}##

Could someone check my work ?
 

Attachments

  • rod.png
    rod.png
    9.1 KB · Views: 442
Physics news on Phys.org
  • #2
It all looks ok to me.
 
  • Like
Likes Vibhor
  • #3
According to official answer key , T=Mg/2 and α = 3g/2L :oldeyes: .

Do you mind rechecking the constraint equation ?

Is there some sign error ?
 
Last edited:
  • #4
Now consider a modified setup .
?temp_hash=3e915a2f9a266e4bf1f683613c191957.jpg

The same four equations in this case would be

##Mg - Tsin2\theta = Ma_y##

##Tcos 2\theta = Ma_x ##

##Tsin \theta \frac{L}{2} = \frac{ML^2}{12}\alpha##

##a_y cos(90°-2\theta) - \frac{\alpha L}{2}cos(90°- \theta) - a_x cos2\theta = 0 ##

Please check the above equations .
 

Attachments

  • modified.jpg
    modified.jpg
    5.3 KB · Views: 353
  • #5
I rechecked my calculations but the result is same as the one in OP .

The answer key doesn't have too many mistakes .
 
  • #6
I also agree with the answer in the OP.

One way to get their answer is to replace the correct constraint equation with the incorrect equation ##a_y = \alpha L/2## (as though the rod is instantaneously rotating about a fixed left end).
 
  • Like
Likes Vibhor
  • #7
(as though the rod is instantaneously rotating about a fixed left end).

Or in the case when both the strings are vertical .

Do you think my equations for the set up in post 4 are correct ?
 
  • #8
Vibhor said:
Or in the case when both the strings are vertical .
Yes, a vertical string would give a constraint of ##a_y = \alpha L/2##. But then, I think you would get T = Mg/4.

Do you think my equations for the set up in post 4 are correct ?
Yes, they look correct to me.
 
  • Like
Likes Vibhor
  • #9
TSny said:
Yes, a vertical string would give a constraint of ##a_y = \alpha L/2##. But then, I think you would get T = Mg/4.

Yes .

TSny said:
Yes, they look correct to me.

Thank you very much .

Thanks @haruspex
 
  • #10
Please see the attached image .

Just after the right string is cut , the net acceleration of the CM of the rod is vertically downwards .Hence rod must move such that CM falls straight down.The left end must also fall straight down .

But net acceleration of the left end point will be towards left ,which means the next instant after right string snaps , it moves towards left .

Aren't the movement of CM and left end contradicting each other .

I am sure I am messing somewhere .
 

Attachments

  • rod2.png
    rod2.png
    11.8 KB · Views: 436
  • #11
Vibhor said:
The left end must also fall straight down .
Why? There will be an angular acceleration.
 
  • Like
Likes Vibhor
  • #12
Right . So the next instant , rod moves such that CM moves straight down and the left end moves leftwards ?
 
  • #13
Vibhor said:
Right . So the next instant , rod moves such that CM moves straight down and the left end moves leftwards ?
Yes.
 

1. What is angular acceleration?

Angular acceleration is the rate of change of an object's angular velocity over time. In other words, it measures how quickly the object is rotating.

2. How is angular acceleration calculated?

Angular acceleration is calculated by dividing the change in angular velocity by the change in time. It is typically represented by the Greek letter alpha (α) and has units of radians per second squared (rad/s²).

3. What factors affect the angular acceleration of a rod?

The angular acceleration of a rod can be affected by several factors, including the applied torque (force that causes rotation), the moment of inertia (resistance to rotation), and the distribution of mass along the rod.

4. How does angular acceleration differ from linear acceleration?

Angular acceleration measures the change in rotational motion, while linear acceleration measures the change in straight-line motion. Additionally, angular acceleration is measured in radians per second squared (rad/s²), while linear acceleration is measured in meters per second squared (m/s²).

5. Can the angular acceleration of a rod be negative?

Yes, the angular acceleration of a rod can be negative if the rod is slowing down or rotating in the opposite direction. This is represented by a negative value for alpha (α) in the equation for angular acceleration.

Similar threads

  • Introductory Physics Homework Help
Replies
7
Views
313
Replies
1
Views
1K
  • Introductory Physics Homework Help
Replies
4
Views
1K
  • Introductory Physics Homework Help
3
Replies
97
Views
3K
  • Introductory Physics Homework Help
Replies
13
Views
1K
  • Introductory Physics Homework Help
2
Replies
38
Views
2K
  • Introductory Physics Homework Help
Replies
25
Views
2K
  • Introductory Physics Homework Help
Replies
3
Views
956
  • Introductory Physics Homework Help
Replies
15
Views
1K
  • Introductory Physics Homework Help
Replies
4
Views
954
Back
Top